4 svar
54 visningar
Cien 1138
Postad: 22 apr 2023 14:15 Redigerad: 22 apr 2023 14:21

Vektorvärd funktion, hastighet

Skulle behöva lite hjälp med både a och b.

Om vi börjar med a.
Jag deriverade r(t) för att få hastigheten, sen satt jag den lika med vektorn som står i (a). Dvs
-2tsint2,2tcost2,1-t2 =(-π2,π2,3π/4)\left( -2tsin \left( t^2 \right) , 2tcos \left( t^2 \right), 1-t^2  \right)=(-\sqrt{ \dfrac{\pi}{2}},\sqrt{\dfrac{\pi}{2}}, 3 \pi /4). Tänkte sen att vi måste få fram tiden t så jag löste ut t till 1-t2=3π/41-t^2=3 \pi /4 vilket blir t=4-3π/2t=\sqrt{\left( 4-3 \pi \right) } /2 vilket jag nu i efterhand ser inte kan stämma då vi får ett negativt tal under roten. Jag testade även att lösa ut t ur -2tsint2=-π2-2tsin\left( t^2 \right)=-\sqrt{ \dfrac{\pi}{2}} men jag kom inte på hur den löses. Hur ska jag göra?

I (b) uppgiften svarade jag som integralen ni ser i bilden, räknade inte ut svaret. Borde det räcka tro?

D4NIEL 2602
Postad: 23 apr 2023 16:16 Redigerad: 23 apr 2023 16:27

a) Förmodligen något tryckfel, tiden liksom positionen blir imaginär vilket känns lite krystat.

b) Visa dina räkningar, dr=(1+t2)dt\|d\mathbf{r}\|=(1+t^2)dt på intervallet 0t30\leq t\leq 3 och integralen som söks borde vara γdr\displaystyle \int_\gamma \left|d\mathbf{r}\right|, vilken är lätt att beräkna.

Cien 1138
Postad: 23 apr 2023 16:40 Redigerad: 23 apr 2023 16:41
D4NIEL skrev:

a) Förmodligen något tryckfel, tiden liksom positionen blir imaginär vilket känns lite krystat.

b) Visa dina räkningar, dr=(1+t2)dt\|d\mathbf{r}\|=(1+t^2)dt på intervallet 0t30\leq t\leq 3 och integralen som söks borde vara γdr\displaystyle \int_\gamma \left|d\mathbf{r}\right|, vilken är lätt att beräkna.

I (b) så tog jag t=03|drdt|dt

Integranden blir (2tsin(t2))2+(2tcos(t2))2+(1-(t2))2\sqrt{(2tsin(t^2))^2+(2tcos(t^2))^2+(1-(t^2))^2} som efter lite förenkling blir t4+2t2+1\sqrt{t^4+2t^2+1} ser nu att jag kan kvadrera bort roten

Cien 1138
Postad: 23 apr 2023 16:47 Redigerad: 23 apr 2023 16:52
D4NIEL skrev:

a) Förmodligen något tryckfel, tiden liksom positionen blir imaginär vilket känns lite krystat.

b) Visa dina räkningar, dr=(1+t2)dt\|d\mathbf{r}\|=(1+t^2)dt på intervallet 0t30\leq t\leq 3 och integralen som söks borde vara γdr\displaystyle \int_\gamma \left|d\mathbf{r}\right|, vilken är lätt att beräkna.

Så är mitt svar korrekt då? för 4-3π/2\sqrt{4-3 \pi }/2 är ju imaginärt. Det var en uppgift på tentan så undrar om jag fick något poäng där 

Cien 1138
Postad: 24 apr 2023 12:04
D4NIEL skrev:

a) Förmodligen något tryckfel, tiden liksom positionen blir imaginär vilket känns lite krystat.

b) Visa dina räkningar, dr=(1+t2)dt\|d\mathbf{r}\|=(1+t^2)dt på intervallet 0t30\leq t\leq 3 och integralen som söks borde vara γdr\displaystyle \int_\gamma \left|d\mathbf{r}\right|, vilken är lätt att beräkna.

Fick lösningsförslag nu och uppgiften var mycket riktigt felformulerad. 

Svara Avbryt
Close